tod has 20 books he donated 2/5 how many books did he donate?

Answers

Answer 1

Given:

Total number of books is, N = 20.

Number of books donated is, d = 2/5 of books.

The objective is to find the number of books donated.

Consider the number of books donated as x.

[tex]\begin{gathered} x=\frac{2}{5}\cdot20 \\ x=8 \end{gathered}[/tex]

Hence, the number of books donated is 8.


Related Questions

I need help with this practice problem solving in trigonometry

Answers

The secant function is the reciprocal of the cosine function.

We can evaluate trigonometric functions of angles outside the first quadrant using reference angles as we have already done with the sine and cosine functions. The procedure is the same:

Find the reference angle formed by the terminal side of the given angle with the horizontal axis. The trigonometric function values for the original angle will be the same as those for the reference angle, except for the positive or negative sign, which is determined by x– and y-values in the original quadrant. (Figure) shows which functions are positive in which quadrant.

An even function is one in which f(-x) = f(x)

An even function is one in which f(-x) = - f(x)

Cosine and secant are even:

[tex]\begin{gathered} \cos (-t)=\cos (t) \\ \sec (-t)=\sec (t) \end{gathered}[/tex]

Therefore from the drop down the correct answer : secant function is EVEN

Choose the best estimate for the quotient. - See image inserted

Answers

Answer:

≈0.694 or rounded to 0.7

Step-by-step explanation:

4.06/5.85≈0.694

Which of the following scatterplots do not show a clear relationship and would not have a trend line?
Group of answer choices

Answers

The scatterplot that do not show a clear relationship and would not have a trend line is: graph B.

What is a trend line?

A trend line is also referred to as a line of best fit and it can be defined as a statistical or analytical tool that is commonly used in conjunction with a scatter plot, in order to determine whether or not there's any association (correlation) between data points.

In Mathematics, there are different types of trend line and these include the following:

Linear trend line.Exponential trend line.Polynomial trend line.Logarithm or Natural logarithm trend line.Power trend line.Moving average trend line.

By critically observing the scatter plot (see attachment) which models the data in the given table, we can reasonably and logically deduce that the data points in graph B are scattered around and as such, it does not illustrate a clear relationship (pattern).

Read more on trend line here: https://brainly.com/question/27194207

#SPJ1


An FICA tax of 7.51% and an SDI deduction of 1.2% are charged against
an employee's wages. If her total monthly wages were $1538.46, what
would her paycheck be after FICA and SDI deductions as well as
withholding tax deductions of $245.80?

Answers

The employee's paycheck after FICA and SDI deductions as well as

withholding tax deductions is $1158.65.

What is FICA?

FICA is a federal payroll tax in the US. It is taken out of every paycheck and stands for the Federal Insurance Contributions Act. Your nine-digit number enables Social Security to precisely record your self-employment or insured wages. You accrue credits for Social Security benefits as you work and pay FICA taxes.

Employee monthly salary= $1538.46FICA calculation= 7.51% of $1538.46= $115.54SDI calculation= 1.2% of $1538.46= $18.47Employee final paycheck= Monthly wage - FICA - SDI - Tax deductions.$1538.46 - $115.54 - $18.47 - $245.80= $1158.65

The final monthly paycheck of the employee after deducting all taxes is $1158.65.

To learn more about FICA, refer to,

https://brainly.com/question/24049080

#SPJ13

Write in simplest form 0.875:1/4

Answers

Hey there!

[tex]0.875:1/4\/\\\textmd{To Simplest form}[/tex]

[tex]0.875[/tex] ÷ [tex]\frac{1}{4}[/tex]

[tex]=3.5[/tex]

Your final answer would be 3.5.

Hope this helps!

0.875:1/4 in simplest form would be 3.5

Using the graphs what are the solutions to the following systems

Answers

The solution to the system is the intersection of the graphs.

From the problem, the intersection is at (-3, 2)

The answer is (-3, 2)

Marlene decided to buy 2 bags of apples weighing 3 14 pounds each, instead of 3 bags weighing 2 25 pounds each. Did she end up with more apples or fewer apples by buying the bigger bags?To find the total weight of 2 bags of apples weighing 3 14 pounds, multiply. 2 × 3 14 = To find the total weight of 3 bags weighing 2 25 pounds, multiply. 3 × 2 25 = She ended up purchasing apples by buying the bigger bags.

Answers

Answer:

She ended up with fewer apples by buying the bigger bags.

Explanation:

The total weight of 2 bags of apples weighing 314 pounds each is equal to:

2 x 314 = 628 pounds

At the same way, the total weight of 3 bags weighing 223 pounds each is:

3 x 225 = 675 pounds

Since 628 pounds is less than 675 pounds, Marlene ended up with fewer apples by buying the bigger bags

If f(x)=x²+5 and g(x)=3x, find (f . g)(x) and (g . f)(x).
(f . g)(x)= ____
(Simplify your answer.)

Answers

The value of the functions are;

(f. g)(x) =  9x² + 5  

(g . f)(x) = 3x² + 15

What is a function?

A function can be defined as a law, rule or expression showing the elaborate relationship between two variables in an expression or equation.

The variables are;

The dependent variableThe independent variable

Given the functions;

f(x)=x²+5g(x)=3x

To determine  (f .g)(x), we substitute the value of g(x) as x  in the function, we have;

(g)(x) = (3x)² + 5

expand the bracket

(g)(x)  = 9x² + 5

To determine (g . f )(x), substitute the value of x as fx)

(g . f)(x) = 3(x² + 5)

expand the bracket

(g . f)(x)= 3x² + 15

Hence, the functions are  9x² + 5 and  3x² + 15

Learn more about functions here:

https://brainly.com/question/25638609

#SPJ4

Which graph represents the following function?

f(x) = |x|.

Answers

The graph of the function f (x) = |x| is shown in attached figure.

Hence, Option A is true.

What is mean by  Modulus function?

The modulus function is the magnitude or absolute value of a positive or negative number. It is defined as y = |x| or f (x) = |x|

Where, f : R → R and x ∈ R.

This is also called the absolute value function.

Given that;

The modulus function is,

⇒ y = |x|

Now, The modulus function is,

⇒ y = |x|

Since, The function y = |x| is not differentiable at x = 0.

So, It break at point x = 0, and have all positive values of y.

Which is shown in image A.

Thus, The graph of function y = |x| is shown in attached figure.

Therefore, In the images;

Option A is true.

Learn more about the modulus function visit:

https://brainly.com/question/28801345

#SPJ1

Which equation could be represented by the number line?

Answers

An equation which could be represented by the number line is: D. -5 + (-1) = -6.

What is a number line?

A number line simply means a type of graph with a graduated straight line which comprises both positive and negative numbers that are placed at equal intervals along its length.

This ultimately implies that, a number line can be used to compare or determine the difference between two numbers. Additionally, a number line typically increases in numerical value towards the right and decreases in numerical value towards the left.

From the number line shown above, we have:

Distance = -5 + (-1)

Distance = -5 - 1

Distance = -6.

Read more on number line here: brainly.com/question/17325155

#SPJ1

K 370 45° Which equation can be used to find the value of k? 37° - K +45º = 180° 37° + k +45º = 180° 37" + k = 45° 180° 37° -45° - 180°

Answers

The sum of angle on a straight line is always 180 degree.

Determine the equation for the angle.

[tex]k+37+45=180[/tex]

The formula V=4/3 pie r3 gives the volume with radius r. Find the volume of a sphere with radius x+9. Write your answer as standard form.

Answers

We will find it as follows:

[tex]V=\frac{4}{3}\pi(x+9)^3\Rightarrow V=\frac{4\pi}{3}(x^3+27x^2+243x+729)^{}[/tex][tex]\Rightarrow V=\frac{4}{3}\pi x^3+36\pi x^2+324x+972[/tex]

The volume of such a sphere is given by the expression found.

***

When we have a binomial at the power of a number, of the form:

[tex](a+b)^n[/tex]

We use Pascal's triangle to determine the expansion. Pascal's triangle is the following:

Each row can be used to calculate a specific power, in our case the binomial is at the power of three, and we will use the 4rth row to expand, that is:

[tex](a+b)^3=a^3+3a^2b+3ab^2+b^3[/tex]

As you can see, the numbers that accompany a & b are the ones found in the 4th row of Pascal's triangle. We also see that the "degree" o the expansion always sums 3, that is a^3 has an overall degree of 3, a^2b has an overall degree of 3, ab^2 has an overall degree of 3 & b^3 has an overall degree of 3.

***

If we, for example, wanted to expand (a + b)^4, we then would have the following:

*We would use the 5th row and get:

[tex](a+b)^4=a^4+4a^3b+6a^2b^2+4ab^3+b^4[/tex]

[Pascal's triangle expands to infinity. So, you in theory can manually expand any binomial at any power, but for big numbers, it will take a long time]

***

In our case, the binomial was:

[tex](x+9)^3[/tex]

And, its respective expansion is:

[tex](x+9)^3=x^3+3x^2(9)+3x(9)^2+9^3[/tex][tex]=x^3+27x^2+243x+729[/tex]

A line passes through the point (-4, 1) and has a slope of -2.
Write an equation in slope-intercept form for this line.

Answers

Answer:

y= -2x -7

Step-by-step explanation:

The equation of a line in slope-intercept form is given by y= mx +c.

m= slopec= y-intercept

Given that the slope is -2, we can substitute m= -2 into the equation.

y= -2x +c

The value of c can be found by substituting a point in which the line passes through into the equation. In this case, the point is (-4, 1).

y= -2x +c

When x= -4, y= 1,

1= -2(-4) +c

1= 8 +c

Subtract 8 on both sides:

c= 1 -8

c= -7

Substituting c= -7 into the equation:

y= -2x -7

Optional step:

To check our answer, we can substitute one coordinate and see if the other coordinate is the same as the given coordinate. For example, let's substitute the x-coordinate of -4 into the equation.

y= -2(-4) -7

y= 8 -7

y= 1

Since the y-coordinate is indeed 1, the equation above is correct.

Additional reading:

For a similar question on slope-intercept form, do check out: https://brainly.com/question/28007941

The half-life of silicon-32 is 710 years. If 50 grams is present now, how much will be present in 700 years?(Round your answer to three decimal places.)A) OB) 25.245C) 0.054D) 46.697

Answers

In this type of question, we will use the exponential equation

[tex]y=a(b)^x[/tex]

Where:

a is the initial amount

b is the factor of growth or decay

Since it is a half-life, then

b = 1/2 = 0.5

Since it is now 50 grams, then

a = 50

Since x is the number of years, then

x = 700

Substitute them in the rule above

[tex]\begin{gathered} y=50(0.5)^{700} \\ y=0 \end{gathered}[/tex]

It will be representing 0 grams after 700 years

The answer is A

Mal works at a photo gallery. He charges $50 for a large photo and $40 for a large frame. Sales tax is 5%. How much total tax will a customer pay on both?
Answer the questions to show how to write and simplify expressions that represent the problem.
1. One way to calculate the total tax is 0.05(50 + 40). Use the order of operations to evaluate this expression. Show your work. (2 points)















2. Use the distributive property to expand 0.05(50 + 40). (2 points)















3. Find the total tax using the expanded expression. Did you get the same answer as you did by evaluating the original expression? (3 points)















4. Which expression, the original expression or the expanded expression, involves finding the total cost first, then calculating the tax on that total? (3 points)

Answers

1. Using the order of mathematical operations, the values in brackets are added and multiplied to a product of $4.50 as the total tax.

2. Using the distributive property to expand the expression, 0.05(50 + 40) is  0.05 x 50 +  0.05 x40.

3. The total tax using the expanded expression is $4.50, the same product as when evaluating the original mathematical expression.

4. The original expression involves finding the total cost first before calculating the tax on the total.

What is the order of mathematical operations?

The following is the order of mathematical operations:

ParenthesesExponentsMultiplicationDivision Addition Subtraction.

1) The original expression, 0.05(50 + 40):

= 0.05 x 90

= 4.5

= $4.50

2) The expanded expression, 0.05(50 + 40)

=  0.05 x 50 +  0.05 x40

= 2.5 + 2.0

= 4.5

= $4.50

Learn more about mathematical operations at https://brainly.com/question/20628271

#SPJ1

(2a) Multiple Choice: Place the letter of the correct answer on the answer line to the right:
For which of the following values of k will the equation have infinite solutions?
4(4x + 1) = k(8x + 2)
(a) 4
(b) 8
(c) 2
(d) 1
(2b) Using your answer from Part (2a) replace the value you selected in the equation below. Then solve the
equation and show that it will, in fact, yield infinite solutions.
4(4x+1)=k(8x + 2)

Answers

For k=2 for infinite solutions.

What is Equation?

Equations are mathematical statements with two algebraic expressions flanking the equals (=) sign on either side. It demonstrates the equality of the relationship between the expressions printed on the left and right sides. LHS = RHS is a common mathematical formula.

Coefficients, variables, operators, constants, terms, expressions, and the equal to sign are some of the components of an equation. The "=" sign and terms on both sides must always be present when writing an equation.

Given:

4(4x + 1) = k(8x + 2)

Now, the equation should have infinite solutions

In consider the situation then we must put value of k such that LHS part become Equal to RHS.

Now, take k=2

4(4x + 1) = k(8x + 2)

LHS: 4(4x+1)= 16x+ 4

RHS: 2(8x+ 2)= 16x+ 4

Hence, k=2 for infinite solutions.

Learn more about equation here:

https://brainly.com/question/10413253

#SPJ1

Choose the point which shows the correct location for the polar coordinate (3, -45°)

Answers

The argument of the polar coordinate is -45 degrees, which can be converted as,

[tex]\begin{gathered} \theta=360^{\circ}-45^{\circ} \\ =315^{\circ} \end{gathered}[/tex]

Thus, the required point is D.

When twenty is reduced by one-third of a number, the result is 19. Find the number.

Answers

Answer:

x = 3

Step-by-step explanation:

When twenty is reduced by one-third of a number, the result is 19. Find the number.

20 - 1/3x = 19

subtract 20 to both sides:

20 - 1/3x - 20 = 19 - 20

-1/3x = -1

multiply both sides by -3:

-3(-1/3x) = -1(-3)

x = 3


The Outsiders
Movie vs. Book Comparison Project

Answers

A book sometimes has less characters while in a movie they introduce new characters

determine if the following sequence is Arithmetic if so what is the common difference -6 -2 2 6

Answers

We are given the following series

-6, -2, 2, 6

Common difference = last term - first term

For the first two terms

The common difference is calculated as

Last term - first term

Last term = -2

First term = -6

Common difference = -2 - (-6)

= -2 + 6

common difference = 4

Common difference = 2 - (-2)

common difference = 2 + 2

Common difference = 4

Last two terms

common difference = 6 - 2

Common difference = 4

The common difference for this sequence is 4

The sequence is arithmetic.

I dont understand the part "Would this be true if the numbers were-3, 5, -16, and -10?"

Answers

Yes, the numbers will be similar.

it will be true if the numbers were -3, 5, -16 and -10

Explanation:

The first list: -3, 5, 16 and -10

Arranging from least to greatest:

-10, -3, 5, 16

Arranging the same numbers from closest to zero to farthest to zero:

Let's use zero as our reference. Then start arranging towards the left and right side of it.

-10, -3, 0, 5, 16

Looking at the above: when we remove 0, we would have same answer as the arrangement from least to greatest:

-10, -3, 5, 16

The second list:

-3, 5, -16, -10

Arranging from least to greatest:

-16, -10, -3, 5

Arranging the same numbers from closest to zero to farthest to zero:

Let's use zero as our reference. Then start arranging towards the left and right side of it.

-16, -10, -3, 0, 5

Looking at the above: when we remove 0, we would have same answer as the arrangement from least to greatest:

-16, -10, -3, 5

Hence, fromt the above: if we arrange the same numbers from least to greatest and from closest to zero to farthest to zero, we would arrive at the same result.

Create a rational function that meets the following criteria.Vertical Asymptotes at x = 5 and x = -3Hole in the graph at x = -1Horizontal Asymptote at y = 7Leave your answer in factored form (do not multiply the factors out)

Answers

[tex]\begin{gathered} \text{ A vertical asymptote implies a denominator of a function where x=0 therefore} \\ x=5\rightarrow x-5=0\rightarrow(x-5) \\ x=-3\rightarrow x-3=0\rightarrow(x-3) \\ \text{ A hole in the graph at x=-1 implied that the factor is both on numerator and denominator} \\ \text{ Therefore,} \\ x=-1\rightarrow(x+1)\text{ is both on numerator and denominator} \\ \text{ A horizontal asymptote at y=7 means the numerator and denominator} \\ \text{ have their coefficients at 7:1} \\ \text{ Putting it together we have} \\ \frac{7(x+1)}{1(x-5)(x-3)(x+1)} \\ \text{simplify and we get }\frac{7(x+1)}{(x-5)(x-3)(x+1)} \end{gathered}[/tex]

Justin has $7.50 more than Eva, and Emma has $12 less than Justin. Together, they have a total of $63.00. Write an equation to determine how much money each person has.

Answers

let j be justin, e for Eva and m for Emma. We get

[tex]\begin{gathered} j+e+m=63 \\ j=e+7.5 \\ m=j-12=e+7.5-12=e-4.5 \end{gathered}[/tex]

so we get that, replacing all the variables

[tex]\begin{gathered} e+7.5+e+e-4.5=63 \\ 3e+3=63 \\ 3e=60 \\ e=20 \end{gathered}[/tex]

so we get that Eva has $20. therefore Justin has $27.5 and Emma has $15.5

I solved this and think the range is: y>=3.Did I solve correctly?

Answers

Notice that

[tex]\begin{gathered} x^2\ge0,x\in\Re \\ \Rightarrow x^2+3\ge3,x\in\Re \\ \Rightarrow f(x)\ge3,x\in\Re \end{gathered}[/tex]

Therefore, the range of the function is f(x)>=3, or [3,infinite)

Write two decimals that are equivalent to the given decimal 3,200

Answers

The two equivalent expression of the decimal 3.200 is 3200/1000 and 3.2

What is defined as the decimals?A decimal is a number with a whole as well as a fractional part. Decimal numbers are between integers as well as represent numeric values for whole plus some fraction of a whole. When we divide a whole in to the smaller parts, we get decimals. A decimal number has two parts: a whole number component and a fractional component. The decimal place value system is identical to the whole number value system for the entire part of a decimal number. However, as we move to the right after the decimal point, we get the fractional part of a decimal number.

For the given question, the stated decimal is;

3.200

Removing point will be;

3200/1000 is the equivalent expression.

Divide numerator an denominator by 100 .

32/10 = 3.2 is also the equivalent expression of 3.200.

Thus, the two equivalent expression of the decimal 3.200 is 3200/1000 and 3.2.

To know more about the decimals, here

https://brainly.com/question/1284106

#SPJ13

The correct question is-

Write two decimals that are equivalent to the given decimal 3.200

Find the result when 60 is increased by 40%.

Answers

Given:

The number is 60.

Aim is to find the result when 60 is increased by 40%.

First calculate 40% of 60,

[tex]\frac{40}{100}\times60=24[/tex]

Now, add the amount 24 in 60.

[tex]24+60=84[/tex]

Answer: 84 is a 40% increase of 60.


Benny had 49 books. He sold
15 at a garage-sale. He used
the money he earned to buy 7
more books. How many books
does Benny have now?

Answers

Answer: 41

so he has 49 books and he sold 15 so its 49-15= 34 so he bought 7 more books so its 34+7=41

Step-by-step explanation:

Have great day

30 locusts eats 420g of grass in a week ,how many days will 21locust take to eat 420g at the same rate .
420/30=14g
14g =2g why is 14g =2g

Answers

It will take 4.9 days for 21 locust to eat 420 grams of grass

How to determine the number of days?

From the question, we have the following parameters:

Initial number of locust = 30Initial number of days = 7 days i.e. 1 weekNew number of locust = 21

Given that the amount of grass is constant at 420 grams

The given parameters can be represented using the following ratio

Ratio = Locust : Days

So, we have

Locust : Days = 30 : 7

When there are a total number of 21 locust eating the grass, then we have

21 : Days = 30 : 7

Express as fraction

Days/21 = 7/30

This gives

Days = 21 * 7/30

Evaluate

Days = 4.9

Hence, the number of days is 4.9 days

Read more about ratio at

https://brainly.com/question/2784798

#SPJ1

As the value of cosx approaches 1 and the value of sinx approaches 0, the value of cotx approaches infinity. True or false?

Answers

Recall that:

[tex]\cot \text{ x=}\frac{\cos x}{\sin x}[/tex]

Therefore as sin x approaches 0 and cos x approaches 1, the value of cot x approaches infinity.

Answer: True.

Great evening to you all, I need help with this math problem please

Answers

AA (or AAA) or Angle-Angle Similarity

If any two angles of a triangle are equal to any two angles of another triangle, then the two triangles are similar to each other.

We do not have 3 angles.

SAS or Side-Angle-Side Similarity

If the two sides of a triangle are in the same proportion of the two sides of another triangle, and the angle inscribed by the two sides in both the triangle are equal, then two triangles are said to be similar.

We have two sides and 1 angle. But, two sides of a triangle are not in the same proportion of the two sides of another triangle

SSS or Side-Side-Side Similarity

If all the three sides of a triangle are in proportion to the three sides of another triangle, then the two triangles are similar.

Letter D

Other Questions
A 5-gallon radiator is full and contains a 40% solution of antifreeze. how much needs to be drained out and replaced with pure antifreeze to obtain a 70% solution? A) The linear model represents the height, f(x), of a water balloon thrown off the roof of a building over times, x measured in seconds. During what interval(s) of the domain is the water balloon's height staying the same?A.0 x 2B.2 x 5C.5 x 6D.6 x 8B) The linear model represents the height, f(x), of a water balloon thrown off the roof of a building over time, x, measured in seconds. During what interval(s) of the domain is the water balloon's height increasing?A.0 x 2B.40 y 70C.5 x 8D.40 y 10C) The linear model represents the height, f(x), of a water balloon thrown off the roof of a building over time, x, measured in seconds. During what interval(s) of the domain is the water balloon's height decreasing the fastest?A.5 x 9.5B.8 x 9.5C.6 x 8D.5 x 6D) The linear model represents the height, f(x), of a water balloon thrown off the roof of a building over time, x, measured in seconds. Justify your answer from Part C.A.5 x 9.5 is the interval where the balloon's height is decreasing.B.8 x 9.5 is the interval where the slope is the steepest.C.6 x 8 is the interval where the balloon's height decreases the most.D.5 x 6 is the interval where the slope is the steepest.Please help as fast as possible How would you explain allocative efficiency in a purely competitive market structure? How does the speaker attempt to prove hisargument's validity?by mentioning people he knows who have sufferedby detailing how these situations often unfoldby making a comparison to an uncontrollable fireby explaining how the problem can be remedied Apple PickingNamejosiphLinda and Robin went apple picking. After picking all day, Linda had 15 moreapples than Robin. Together, they gathered 105 apples. Figure out how manyapples Linda got and how many Robin got. Linda got 1/3 of all her apples fromthe first tree and % of all her apples from the second tree. She got the rest of herapples from the third tree. Robin got 1/3 of all her apples from the first tree and5/9 of all her apples from the second tree. She got the rest of her apples fromthe third tree. Who got more apples from the third tree? How many more? A flying carpet flies 2.4 miles with the wind in the same amount of time it flies 1.4 miles against the wind. The wind speed is 4 mph what is the speed of the flying carpet A person randomly selects one of six envelopes Each envelope contains a check that the person gets to keep. Determine the person's expectation if three envelopes contain a 491 check and three envelopes contain a 51003 checkThe expected value is $(Simplify your anwwer Type an integer or a decimal) On a coordinate plane, a graph decreases between (2.5, 2) and (4.5, 0.25).How does this graph change between (2.5, 2) and (4.5, 0.25)? the sixth amendment is the basis for the righta) to a delayed trial.b) of free speech.c) of assembly.d) to counsel Why did people believe in the hitler Youth? write 70 1/3% as a fraction in simplest form.How to get the answer is the main question for me. I got it wrong on the assignment. stranded wire use is being discussed. technician a says there is less exposed surface area for electron flow in stranded wire. technician b says there is more resistance in the stranded wire than in the same gauge solid wire. who is correct? why is it so common in internet protocols that clients speak before servers do? (in other words, what would be an important advantage of having a client speak first, or, thought differently, an important disadvantage of having a server speak first instead?) (24 3) x 4-(30 3) Rafael wants to construct a square inscribed in a circle. He divides the circle into six equal arcs and then draws line segments between four ofthe points. What was the first error Rafael made?:toThe first error Rafael made wasV. A square is constructed by first Which of the following structures in recombinant DNA contains unpaired overhanging nucleotides after DNA is cut with a restriction enzyme?A. DNA libraryOB. Sticky endsOC. Blunt endsOD. DNA ligase In 2017 the population of Rexburg, Idaho was 28,337 people. The population was expected to grow at a rate of about 1.21% per year. Based on these numbers, what would we predict the population of Rexburg will be in the year 2020? What is denail or mount mckinley about? You visit your favorite restaurant and order an appetizer for $5.99, a drink for $4.50, your meal for $10.99, and dessert for $3.75. The sales tax is 9%. and youwould like to leave an 18% tip. Find the total cost of your meal. What does like a trees growing on that side?